LSAT and Law School Admissions Forum

Get expert LSAT preparation and law school admissions advice from PowerScore Test Preparation.

User avatar
 Dave Killoran
PowerScore Staff
  • PowerScore Staff
  • Posts: 5972
  • Joined: Mar 25, 2011
|
#72531
Complete Question Explanation

Weaken-SN. The correct answer choice is (D)

This is another difficult problem but very typical of the LSAT. As always, the key to success is to isolate the conclusion, which appears in the last sentence: “companies producing carpet will be able to gain market share in the carpet market only through purchasing competitors.” As you should have noted while reading, the conclusion contains a conditional indicator and is thereby conditional in nature. The conclusion can be diagrammed as:

  • GMS = gain market share in the carpet market
    PC = purchasing competitors


    GMS :arrow: PC
According to the author, to gain market share in the carpet market a company must purchase a competitor. Answer choice (C) is often selected by students, but it does not attack this idea. To attack a conditional statement you must show that the necessary condition is not actually necessary for the sufficient condition to occur. Answer choice (C) simply suggests that when companies purchase their competitors the endeavor is often financially unsuccessful. Essentially, answer choice (C) fails to prove that purchasing competitors is unnecessary to gain market share. Answer choice (D), on the other hand, does suggest a way for companies to gain market share without purchasing competitors, thereby attacking the conditional statement given in the stimulus. Thus, answer choice (D) is correct.

Answer choice (A): This answer goes beyond the scope of the argument, which is limited to the carpet market (and not other floor coverings).

Answer choice (B): This is an Opposite answer that strengthens the argument. If there are no remaining niches to fill, then there is no way to expand other than to purchase a competitor.

Answer choice (C): This attractive answer is wrong for two very strong reasons:

  • 1. A Shell Game is played with the details of the conclusion. The conclusion is about market share. Answer choice (C) is about a decline in profits and revenues. The two are not the same, and so the information in the answer choice does not weaken the conclusion.

    2. Even if you assume that market share is the same thing as profits and revenues, a second Shell Game is played because the answer then attacks a conclusion that is similar but different from the given conclusion.

    If the conclusion were as follows:

         PC :arrow: GMS

    Then answer choice (C) would be correct (again, assuming market share is the same thing as profits and revenues). But, the above representation is a Mistaken Reversal of the conclusion, and so the attack is made on a statement that uses the same terms as the conclusion but puts them in a different relationship. Stop for a moment and think about how this works, because this is a great example of the cleverness displayed by the test makers. They’ve presented an answer that would effectively attack a conclusion that is very similar to the actual conclusion in the argument (but that is in fact different). Fortunately you can avoid this answer if you know what to look for when attacking conditional reasoning.
One point worth noting is that it is no accident that the most tempting wrong answer choice appears just before the correct answer. This is a classic LSAT trick, and one that is very effective because most test takers relax once they find an answer they feel is attractive. This makes them less likely to closely examine the answers that follow. Never relax during the LSAT!

Answer choice (D): This is the correct answer. If price reductions drive out some of the carpet producers, then other producers can take the market share left behind. This scenario shows that a company can gain market share without purchasing a competitor, thus attacking the necessary condition in the conclusion.

Answer choice (E): This Opposite answer strengthens the argument. If the consumers are resistant to new styles, then one fewer possibility exists if a company is trying to increase market share. By eliminating this option, the conclusion is strengthened (if you eliminate an idea that would hurt the argument, that strengthens the argument because it has fewer “competitors”).
 biskam
  • Posts: 124
  • Joined: Aug 18, 2017
|
#40728
I chose d but can someone please tell me why c is incorrect? Is it because the argument is only saying that purchasing competitors is the only way to gain market share--it's not guaranteeing that purchasing competitors will be effective/profitable?

Thanks!
User avatar
 Dave Killoran
PowerScore Staff
  • PowerScore Staff
  • Posts: 5972
  • Joined: Mar 25, 2011
|
#40740
Hi Biskam,

Remind me if you can of which of our materials you have—there is a detailed explanation of this question in both the LSAT Course books and the LRB, and I' want to direct you to those first because the explanation of this answer will be useful to you.

Thanks!
 biskam
  • Posts: 124
  • Joined: Aug 18, 2017
|
#40775
I took the online course! I have the class books
User avatar
 Dave Killoran
PowerScore Staff
  • PowerScore Staff
  • Posts: 5972
  • Joined: Mar 25, 2011
|
#40776
biskam wrote:I took the online course! I have the class books

I thought so! Ok, please go to Lesson 3 Homework, page 3-76. Check out 3-76 through 3-78 and let me know if that explains this one. One of my favorite questions of all time!
 rimasu
  • Posts: 9
  • Joined: Apr 10, 2018
|
#44992
P 244- 247.

Hello.
I understand this question generally, but some parts written in this book are so complicated that I don't get it at all.
First, according to the last part of right side annotation, in p247 of the 2017 LRB, it says 'answer choice attacks not the real conclusion, but a Reversal of the conclusion', and I understood the Janelle's case attacks the first statement with this logic.
Original: Top :arrow: stairs
Reversal: stairs :arrow: Top
attacks: stairs(j) :arrow: NO Top(j)

But this logic don't apply to carpet market case in this sense.
The original one's diagram is GMS :arrow: PC. , and its reversal one is PC :arrow: GMS.
So if we want to attack original conclusion, we should use PC :arrow: NO GMS , and that one is (c). p244.
The correct answer is (d), which could be diagrammed as NO PC :arrow: GMS.

Thus according to a logic introduced in p247 side bar, it is not suited for this question.
Did I make something wrong?
Could you explain me in detail?
I have struggled with this question for 2 days.

Thanks.
User avatar
 Jonathan Evans
PowerScore Staff
  • PowerScore Staff
  • Posts: 727
  • Joined: Jun 09, 2016
|
#45019
Hey Rimasu,

Very good questions! Let's see whether we can illustrate the problem with Answer Choice (C) and also further illuminate the question in general.

What is the author arguing? The author concludes that carpet makers are only going to get more market share from purchasing competitors.

What's our task? Our job is to show that he's wrong: there might be another way to get more market share other than purchasing competitors.

This is why we have a problem with answer choice (C). (C) says we purchased competitors but instead of making more money, we're doing worse. Well, that stinks, but it does not attack our conclusion. Our conclusion says that purchasing competitors is the only way to get more market share, but our conclusion does not say this is a sure thing.

The stairs analogy works like this:
  • Our conclusion: If you Get Market Share, then you Purchased Competitors.
    Stairs analogy: If you get to the Top Floor, then you Took the Stairs.

    Answer Choice (C): You Purchased Competitors, but you didn't get more market share.
    Stairs analogy: You took the stairs, but you didn't get to the top floor.
Do you see the problem? Just because you purchased competitors doesn't mean you're guaranteed to get more market share. Just because you took the stairs doesn't mean you're guaranteed to get to the top floor.

This is a Mistaken Reversal™: treating a necessary/required condition as though it were sufficient/a guarantee for something else.

Does this help to clear this up a little bit? Thanks!
 rimasu
  • Posts: 9
  • Joined: Apr 10, 2018
|
#45024
Thanks for detailed explanation. :)
I still want to make sure a few things more.

First,
I made a diagram for (D) as NO PC :arrow: GMS
I analyzed (D) in this way.
"Price reduction" = NO PC
"are casuing" = :arrow:
"other producers to leave the market altogether" = GMS

However, from the conclusion, which is diagrammed as GMS :arrow: PC , (D) should be GMS :arrow: NO PC.
I don't get it how I should make a diagram for (D) like that.

Second,
It's about "opposite' issue I asked you before.
In p 246, the last paragraph, I don't understand why this is opposite.
I thought answer choice (E) is just repeating stimulus; 'Most who purchase carpet do so only once or twice, first in their twenties or thirties, and then again in their fifties or sixties" So obviously it strengthens the argument. Isn't it correct?
So from this, I have a difficulty in getting the phrase "By eliminating this option... ..."
What is "this option"? And which one is eliminated?
And if possible how can I make a diagram for this?
User avatar
 Jonathan Evans
PowerScore Staff
  • PowerScore Staff
  • Posts: 727
  • Joined: Jun 09, 2016
|
#45029
Hi, Rimasu,

Thanks for following up!

I see where you're going with your diagramming, but you're getting a little side-tracked with some of your work here. Let's recap what the conclusion says, what the answer choices say, and what we're trying to do.
  • Conclusion: Gain Market Share only by Purchasing Competitors. GMS :arrow: PC

    What we're trying to do: Weaken this conclusion.

    How do we weaken this conclusion? Show that Gaining Market Share can occur without Purchasing Competitors. We are trying to show that the necessary condition (Purchasing Competitors) is not necessary after all.
The main issue in your diagramming for (D) is that "are causing" is not the same as :arrow:.

Price Reductions could indeed be interpreted as "not Purchasing Competitors"/"PC. However, (D) has not shown that Price Reductions guarantee Gaining Market Share. All answer choice (D) has shown is that it's possible to Gain Market Share without Purchasing Competitors. We don't need any arrows here.

Let me explain further:
  • If the conclusion says GMS :arrow: PC, then what would we need to show to hurt this conclusion? All we would need to show is that it's possible to have GMS happen and not PC happen. We don't need a new conditional here. We just need to show that the original conditional is not valid.


Thus, for Answer Choice (D), the only diagramming you'd need is one that says GMS is true and PC is true too. Therefore the original conditional GMS :arrow: PC doesn't check out any more. We don't need to pay attention to any causation here, and we don't need any additional conditional diagramming.

Does this make sense?

For Answer Choice (E), we mean Opposite answer in the sense that it does the opposite of what we want our answer to do. This answer strengthens the argument instead of weakening it, so when you're noting that (E) kinda restates a premise of the argument, you've got it exactly right! This answer choice is backing up the conclusion.

The "option" that Answer Choice (E) is eliminating is that the companies could gain market share through new styles. If we take away the possibility that companies could gain market share with new styles, we have eliminated another way to gain market share and have strengthened the argument that gaining market share is only possible by purchasing competitors.

There is no useful or practicable way to diagram Answer Choice (E); it's not worth the trouble. Be careful not to overkill with diagramming.

I hope this helps!
 rimasu
  • Posts: 9
  • Joined: Apr 10, 2018
|
#45060
Thanks!
I really appreciate your help.
It was a very tricky one for me, but thanks to your explanations, I could finally move on next page!

Ah, one more!
then we don't have to or we can't make all sentences diagrammed?
Only when we could find it is conditional sentece? With words or phrases to introduce conditional reasoning?






Jonathan Evans wrote:Hi, Rimasu,

Thanks for following up!

I see where you're going with your diagramming, but you're getting a little side-tracked with some of your work here. Let's recap what the conclusion says, what the answer choices say, and what we're trying to do.
  • Conclusion: Gain Market Share only by Purchasing Competitors. GMS :arrow: PC

    What we're trying to do: Weaken this conclusion.

    How do we weaken this conclusion? Show that Gaining Market Share can occur without Purchasing Competitors. We are trying to show that the necessary condition (Purchasing Competitors) is not necessary after all.
The main issue in your diagramming for (D) is that "are causing" is not the same as :arrow:.

Price Reductions could indeed be interpreted as "not Purchasing Competitors"/"PC. However, (D) has not shown that Price Reductions guarantee Gaining Market Share. All answer choice (D) has shown is that it's possible to Gain Market Share without Purchasing Competitors. We don't need any arrows here.

Let me explain further:
  • If the conclusion says GMS :arrow: PC, then what would we need to show to hurt this conclusion? All we would need to show is that it's possible to have GMS happen and not PC happen. We don't need a new conditional here. We just need to show that the original conditional is not valid.


Thus, for Answer Choice (D), the only diagramming you'd need is one that says GMS is true and PC is true too. Therefore the original conditional GMS :arrow: PC doesn't check out any more. We don't need to pay attention to any causation here, and we don't need any additional conditional diagramming.

Does this make sense?

For Answer Choice (E), we mean Opposite answer in the sense that it does the opposite of what we want our answer to do. This answer strengthens the argument instead of weakening it, so when you're noting that (E) kinda restates a premise of the argument, you've got it exactly right! This answer choice is backing up the conclusion.

The "option" that Answer Choice (E) is eliminating is that the companies could gain market share through new styles. If we take away the possibility that companies could gain market share with new styles, we have eliminated another way to gain market share and have strengthened the argument that gaining market share is only possible by purchasing competitors.

There is no useful or practicable way to diagram Answer Choice (E); it's not worth the trouble. Be careful not to overkill with diagramming.

I hope this helps!

Get the most out of your LSAT Prep Plus subscription.

Analyze and track your performance with our Testing and Analytics Package.